Help with fourier transform for special square wave

Click For Summary
SUMMARY

The discussion focuses on generating a modified square wave using Fourier analysis, specifically a square wave with longer peaks than valleys. The proposed solution involves rectifying and re-normalizing the wave, leading to the formulation of the wavetrain as rect(x/2) * comb(x/4), where convolution is applied. The Fourier transform of this function is expressed as Sinc(2q) * Sinc(4q), with sinc(x) defined as sin(x)/x and q being the conjugate variable to x. This method effectively reproduces the desired waveform.

PREREQUISITES
  • Understanding of Fourier analysis and its applications
  • Familiarity with the concept of convolution in signal processing
  • Knowledge of the Sinc function and its properties
  • Basic skills in mathematical function manipulation and normalization
NEXT STEPS
  • Study the properties of the Sinc function in signal processing
  • Learn about convolution and its role in waveform generation
  • Explore advanced Fourier transform techniques for non-standard waveforms
  • Investigate rectification and normalization methods for periodic functions
USEFUL FOR

Signal processing engineers, mathematicians, and anyone involved in waveform analysis or synthesis will benefit from this discussion.

xanthium
Messages
2
Reaction score
0
I know how to describe a square wave with Fourier analysis, but what if I'm looking for a square wave with "peaks" that are longer than the "valleys."
For example, from f(x)=1 {from 0 to 2}, f(x)=-1 {from 2 to 3}, f(x)=1 {from 3 to 5}, f(x)=-1 {from 5 to 6}... and so on in a periodic fashion.
Any ideas?
Thanks
 
Physics news on Phys.org
That's not so bad- first, I'll rectify the wave and re-normalize it to make the functions easier to type.

Then, the wavetrain you describe is rect(x/2)*comb(x/4), where '*' is convolution. I'm pretty sure that will reproduce the function you describe. The Fourier transform is then Sinc(2q)*Sinc(4q), where sinc(x) = sin(x)/x, and q is the conjugate variable to x.

Tweak the original function as needed.
 
Question: A clock's minute hand has length 4 and its hour hand has length 3. What is the distance between the tips at the moment when it is increasing most rapidly?(Putnam Exam Question) Answer: Making assumption that both the hands moves at constant angular velocities, the answer is ## \sqrt{7} .## But don't you think this assumption is somewhat doubtful and wrong?

Similar threads

  • · Replies 5 ·
Replies
5
Views
2K
  • · Replies 3 ·
Replies
3
Views
2K
Replies
1
Views
2K
  • · Replies 6 ·
Replies
6
Views
3K
  • · Replies 3 ·
Replies
3
Views
2K
  • · Replies 6 ·
Replies
6
Views
2K
  • · Replies 1 ·
Replies
1
Views
2K
  • · Replies 2 ·
Replies
2
Views
1K
Replies
8
Views
2K
  • · Replies 9 ·
Replies
9
Views
2K